Η αναζήτηση βρήκε 1055 εγγραφές

από kwstas12345
Παρ Μαρ 07, 2014 7:58 pm
Δ. Συζήτηση: Διαγωνισμοί για φοιτητές
Θέμα: SEEMOUS 2014
Απαντήσεις: 5
Προβολές: 2518

Re: SEEMOUS 2014

Και μια λύση για το 4ο,: Γνωρίζουμε ότι $\displaystyle \arctan(x)=\sum_{k=0}^{\infty}{\frac{(-1)^k x^{2k+1}}{2k+1}}, |x|\leqslant 1$, επομένως :$\displaystyle n\int_{0}^{n}{\frac{\arctan\left(\frac{x}{n} \right)}{x\left(1+x^2 \right)}}dx=\int_{0}^{n}{\left(\frac{1}{1+x^2}+\sum_{k=1}^{\infty}{\frac{(...
από kwstas12345
Σάβ Δεκ 07, 2013 12:04 am
Δ. Συζήτηση: ΑΝΑΛΥΣΗ
Θέμα: Ολοκλήρωμα
Απαντήσεις: 2
Προβολές: 659

Re: Ολοκλήρωμα

Αρχικά έχουμε: $\displaystyle \int_{0}^{1}{\ln\left(1+\frac{\ln^2 x}{4\pi^2} \right)\frac{\ln\left(1-x \right)}{x}}dx=2\pi\int_{0}^{\infty}{\ln\left(1+x^2 \right)\ln\left(1-e^{-2\pi x} \right)}dx$ $\displaystyle =-2\pi\sum_{k=1}^{\infty}{\frac{1}{k}\int_{0}^{\infty}{\ln\left(1+x^2 \right)e^{-2\pi kx...
από kwstas12345
Δευ Δεκ 02, 2013 9:18 pm
Δ. Συζήτηση: Διαγωνισμοί για φοιτητές
Θέμα: Όριο με ολοκλήρωμα (1)
Απαντήσεις: 2
Προβολές: 695

Re: Όριο με ολοκλήρωμα (1)

Έχουμε ότι $\displaystyle \left(1+x^n \right)^{1/n}-1=\sum_{k=1}^{\infty}{\frac{\ln^k\left(1+x^n \right)}{n^k}}$ άρα από το θεώρημα Tonelli: $\displaystyle n^2\int_{0}^{1}{\left(\sqrt[n]{1+x^n} -1\right)}dx=n\int_{0}^{1}{\ln\left(1+x^n \right)}dx+\int_{0}^{1}{\ln^2\left(1+x^n \right)}dx+\sum_{k=3}^{...
από kwstas12345
Κυρ Δεκ 01, 2013 5:07 pm
Δ. Συζήτηση: ΑΝΑΛΥΣΗ
Θέμα: Γενικευμένα Ολοκληρώματα
Απαντήσεις: 222
Προβολές: 45028

Re: Γενικευμένα Ολοκληρώματα

66 ) Ας αποδειχθεί οτι $\boxed{\displaystyle{\int_{0}^{\infty}{[(x^n+1)^{\frac{1}{n}}-x]dx}=\frac{1}{2n}B(1-2/n, 1/n)}}$ 67 ) Ας αποδειχθεί οτι $\boxed{\displaystyle{\frac{1}{\pi}{\int_{0}^{\infty}{ln^4(\frac{x}{x^2+1})\frac{dx}{(x^2+1)^2}}}=\frac{19}{960}{\pi}^4+\frac{1}{2}{\pi}^2ln^22+4ln^42+3\ze...
από kwstas12345
Δευ Νοέμ 25, 2013 5:48 pm
Δ. Συζήτηση: ΑΝΑΛΥΣΗ
Θέμα: Πρώτα συνεχής και μετά φραγμένη
Απαντήσεις: 3
Προβολές: 610

Re: Πρώτα συνεχής και μετά φραγμένη

Μια λύση με σειρές. Καταρχάς αφού η $f$ φραγμένη στο $(-1,1)$ έπεται ότι υπάρχει $M>0$ : $\displaystyle \left|f\left(x \right)-f\left(\left[x \right] \right) \right|\leq M\Rightarrow \left|f\left(x \right)-\left[x \right]f\left(1 \right) \right|\leqslant M$ από εδώ έπεται $\displaystyle \sup\left\{\...
από kwstas12345
Σάβ Νοέμ 23, 2013 2:36 am
Δ. Συζήτηση: ΑΝΑΛΥΣΗ
Θέμα: Γενικευμένο ολοκλήρωμα 7.
Απαντήσεις: 3
Προβολές: 978

Re: Γενικευμένο ολοκλήρωμα 7.

Παρατηρούμε ότι ισχύει: $\displaystyle \int_{0}^{\infty}{\frac{\ln(\cos^2 x)}{e^{2x}+1}}dx=-\frac{\ln^2 2}{2}+\frac{1}{2}\int_{0}^{\infty}{\frac{\ln\left(1+\cos x \right)}{e^x +1 }}dx$. Έχει αποδειχθεί οτι $\displaystyle \ln\left(1+\cos(x) \right)=\ln\left(1-\cos\left(\pi -x \right) \right)=-\ln2 -2...
από kwstas12345
Τετ Νοέμ 13, 2013 3:39 pm
Δ. Συζήτηση: ΑΝΑΛΥΣΗ
Θέμα: Γενικευμένο ολοκλήρωμα 9
Απαντήσεις: 2
Προβολές: 696

Re: Γενικευμένο ολοκλήρωμα 9

Θα αποδείξουμε ότι ισχύει: $\displaystyle \ln\left(1-\cos x \right)=-\ln 2 -2\sum_{n=1}^{\infty}{\frac{\cos\left(nx \right)}{n}}, 0<x<\pi, x\neq \pi/2$. Για $r<1$ έχουμε $\displaystyle \ln\left(1-re^{ix} \right)=-\sum_{n=1}^{\infty}{\frac{r^n e^{inx}}{n}}$ οπότε $\displaystyle Re\left(\ln\left(1-re^...
από kwstas12345
Τετ Νοέμ 13, 2013 2:02 pm
Δ. Συζήτηση: ΑΝΑΛΥΣΗ
Θέμα: Σειρά 3.
Απαντήσεις: 6
Προβολές: 971

Re: Σειρά 3.

Ας δούμε άλλη μια λύση για την αρχική: $\displaystyle \sum_{n=1}^{\infty}{\frac{\cos\left(n\pi/3 \right)}{n^2}}=\frac{1}{9}\sum_{n=1}^{\infty}{\frac{\left(-1 \right)^n}{n^2}}+\frac{1}{2}\sum_{n=0}^{\infty}{\frac{\left(-1 \right)^n}{\left(3n+1 \right)^2}}-\frac{1}{2}\sum_{n=0}^{\infty}{\frac{\left(-1...
από kwstas12345
Τετ Οκτ 23, 2013 1:55 pm
Δ. Συζήτηση: ΑΝΑΛΥΣΗ
Θέμα: Επέκταση ολοκληρωμάτων Fresnel.
Απαντήσεις: 3
Προβολές: 842

Re: Επέκταση ολοκληρωμάτων Fresnel.

Αρχικά έχουμε : $\displaystyle \int_{0}^{\infty}{\frac{x^{-1/a}}{x^2 +1}}dx=\frac{1}{2}\int_{0}^{\infty}{\frac{x^{-1/2-1/(2a)}}{x+1}}dx=\frac{1}{2}\int_{0}^{\infty}{\int_{0}^{\infty}{x^{-1/2 -1/(2a)}e^{-x-xy}dydx}}$ $\displaystyle =_{*}\frac{1}{2}\Gamma\left(\frac{1}{2}-\frac{1}{2a} \right)\int_{0}^...
από kwstas12345
Πέμ Οκτ 17, 2013 9:08 pm
Δ. Συζήτηση: ΑΝΑΛΥΣΗ
Θέμα: Γενικευμένο ολοκλήρωμα 5
Απαντήσεις: 2
Προβολές: 616

Re: Γενικευμένο ολοκλήρωμα 5

Έστω $M>0$ τότε έχουμε $\displaystyle \int_{0}^{M}\frac{\sin(at)\cdot \sin t}{t}dt=\frac{1}{2}\int_{0}^{M}{\int_{0}^{\infty}{\left(\cos \left((a-1)t \right)-\cos\left((a+1)t \right) \right)e^{-xt} dx dt}}$ και επειδή $\displaystyle \frac{1}{2}\int_{0}^{M}{\int_{0}^{\infty}{\left|\left(\cos \left((a-...
από kwstas12345
Κυρ Οκτ 13, 2013 9:49 pm
Δ. Συζήτηση: ΑΛΓΕΒΡΑ
Θέμα: Διάσταση χώρου Banach
Απαντήσεις: 3
Προβολές: 1000

Re: Διάσταση χώρου Banach

Συμφωνώ ότι η υπόθεση του συνεχούς κάνει τα πράγματα αρκετά εύκολα, ωστόσο όχι τετριμμένα μιας και χρειάζεται να ξέρει κανείς το θεώρημα Baire για να δείξει ότι ένας πλήρης χώρος έχει υπεραριθμίσιμη βάση Hamel.Παραθέτω ένα σύνδεσμο για την απόδειξη του α: http://user.math.uzh.ch/halbeisen/publicatio...
από kwstas12345
Κυρ Οκτ 13, 2013 6:05 pm
Δ. Συζήτηση: ΑΝΑΛΥΣΗ
Θέμα: Να αντιγράψουμε τον ...Fourier
Απαντήσεις: 9
Προβολές: 1217

Re: Να αντιγράψουμε τον ...Fourier

κ.Σμυρλή νομίζω πως ισχύει αυτό που λέω:Έστω $\displaystyle x=\frac{a}{b}$ τότε αν $\displaystyle \left(p,q \right)\in \mathbb{Z}\times \mathbb{N}:\frac{p}{q}\neq \frac{a}{b}\Rightarrow \left|pb-aq \right|\geqslant 1$ άρα $\displaystyle \left|\frac{a}{b}-\frac{p}{q} \right|\geqslant \frac{1}{bq}$. Α...
από kwstas12345
Κυρ Οκτ 13, 2013 12:53 pm
Δ. Συζήτηση: ΑΝΑΛΥΣΗ
Θέμα: Να αντιγράψουμε τον ...Fourier
Απαντήσεις: 9
Προβολές: 1217

Re: Να αντιγράψουμε τον ...Fourier

Θα χρησιμοποιήσουμε το εξής λήμμα: Άν $\displaystyle x \in \mathbb{Q}\Rightarrow\exists c>0: \forall p,q \in \mathbb{N}$ να ισχύει $\displaystyle \left|x-\frac{p}{q} \right|\geqslant \frac{c}{q}$.Ας υποθέσουμε ότι ο αριθμός μας είναι ρητός ,και έστω $\displaystyle \varepsilon >0\Rightarrow \exists n...
από kwstas12345
Σάβ Οκτ 12, 2013 2:10 am
Δ. Συζήτηση: ΑΛΓΕΒΡΑ
Θέμα: Διάσταση χώρου Banach
Απαντήσεις: 3
Προβολές: 1000

Re: Διάσταση χώρου Banach

Δείχνουμε αρχικά ότιο ο χώρος έχει υπεραριθμήσιμη διάσταση, πράγματι αν αρνηθούμε το ζητούμενο , έστω $\displaystyle \left\{x_{n}:n\in \mathbb{N} \right\}$ μια αριθμήσιμη βάση Hammel.Tότε ορίζοντας $\displaystyle X_{n}:=\span\left\{x_{i}, 1\leq i\leq n \right\}$, έχουμε ότι είναι κλειστοί υπόχωροι (...
από kwstas12345
Τρί Οκτ 08, 2013 12:54 pm
Δ. Συζήτηση: ΑΝΑΛΥΣΗ
Θέμα: Όμορφο γενικευμένο 2.
Απαντήσεις: 2
Προβολές: 809

Re: Όμορφο γενικευμένο 2.

Εύκολα βλέπουμε ότι ισχύει: $\displaystyle \int_{0}^{\infty}\frac{\ln\left(1-e^{-2\pi x} \right)}{x^2 +1}dx=-\sum_{k=1}^{\infty}\frac{1}{k}\int_{0}^{\infty}{\frac{e^{-2k \pi x}}{x^2+1}}dx$. Θέτουμε $\displaystyle I(a):=\int_{0}^{\infty}{\frac{e^{-ax}}{x^2 +1 }}dx$, τότε από τον κανόνα παραγώγισης το...
από kwstas12345
Δευ Οκτ 07, 2013 5:21 pm
Δ. Συζήτηση: ΑΝΑΛΥΣΗ
Θέμα: Όμορφο γενικευμένο 4.
Απαντήσεις: 3
Προβολές: 796

Re: Όμορφο γενικευμένο 4.

Πολύ όμορφο θέμα. Θέτουμε $\displaystyle a_{k}:=\int_{0}^{\pi/2}{x \cos^k x dx}$ τότε από το θεώρημα Beppo-Levi έχουμε $\displaystyle \int_{0}^{\pi/2}{x\ln\left(1-2\cos x \right)}dx=-\sum_{k=1}^{\infty}{\frac{a_{k}}{k}}$.Επίσης έχουμε $\displaystyle a_{k}=\int_{0}^{\pi/2}{x\cos^{k-1}x \left(\sin x \...
από kwstas12345
Τετ Οκτ 02, 2013 2:06 pm
Δ. Συζήτηση: ΑΝΑΛΥΣΗ
Θέμα: Όρια με ολοκληρώματα
Απαντήσεις: 214
Προβολές: 27445

Re: Όρια με ολοκληρώματα

$78)$ Αν $\displaystyle{J_{n} :=\int_0^{\pi}\frac {| \sin (n + 1/2)x|}{x(6 - x)}dx}$, δείξτε ότι $\displaystyle{\lim_{n\to\infty}\frac {J_{n}}{\ln n}=\frac {1}{3\pi}}$. Προφανώς έχουμε $\displaystyle \frac{J_{n}}{\ln n}\geqslant \frac{1}{6}\int_{0}^{\pi}\frac{\left|\sin\left(n+1/2 \right)x \right|}...
από kwstas12345
Τετ Οκτ 02, 2013 12:07 pm
Δ. Συζήτηση: Διαγωνισμοί για φοιτητές
Θέμα: Kyiv Taras 2013 problem 4
Απαντήσεις: 2
Προβολές: 1180

Re: Kyiv Taras 2013 problem 4

Δείχνουμε ότι τουλάχιστον ένας από τους $A,B$ είναι αντιστρέψιμος.Πράγματι αν όχι έστω $r_{1},r_{2}$ οι τάξεις τους που είναι γνήσια μικρότερες του $n$ τότε $\displaystyle A=Q_{1}\begin{pmatrix} I_{r_{1}} & O\\ O & O \end{pmatrix}P_{1},B=Q_{2}\begin{pmatrix} I_{r_{2}} & O\\ O & O \end{pmatrix}P_{2}$...
από kwstas12345
Τρί Σεπ 24, 2013 1:24 pm
Δ. Συζήτηση: Ανάλυση
Θέμα: Όμορφη από Ρουμανία
Απαντήσεις: 5
Προβολές: 1003

Re: Όμορφη από Ρουμανία

Να γράψω λίγο πιο καθαρά γιατι το όριο είναι πραγματικος αριθμός. Άν $\displaystyle x_{0}:f(x_{0})>x_{0}$ τότε $\displaystyle \left(\frac{x_{0}}{f\left(x_{0} \right)} \right)^n\rightarrow 0$ άρα από την αρχή της μεταφοράς έχουμε $\displaystyle \frac{f(x_{0})}{x_{0}}=\lim_{n\rightarrow \infty}\frac{1...
από kwstas12345
Τρί Σεπ 24, 2013 12:13 pm
Δ. Συζήτηση: Ανάλυση
Θέμα: Όμορφη από Ρουμανία
Απαντήσεις: 5
Προβολές: 1003

Re: Όμορφη από Ρουμανία

Και άλλη μια λύση που είχα στείλει χθες στον κ.Σπύρο αλλά ξέχασα να την γράψω: Aρχικά η συναρτησή μας είναι 1-1 και λόγω συνέχειας μονότονη (γνήσιως αύξουσα) και εχει όριο στο άπειρο το άπειρο άρα είναι επί και αντιστρέφεται.Μια συνάρτηση που ικανοποιεί τηνν αρχική σχέση εύκολα προκύπτει ότι ικανοπο...

Επιστροφή στην ειδική αναζήτηση